The passage suggests which one of the following about medical practices in late–nineteenth century Canton, China?

Julie-V on September 6, 2019

Answer Explanation

Hi LSAT Max, Would answer choice (E) be justified by the first sentence of the passage as well as lines 36-38? I got this question wrong but after looking through the passage I think these lines would be the appropriate support. Thanks!

Replies
Create a free account to read and take part in forum discussions.

Already have an account? log in

Julie-V on September 6, 2019

I meant (D), not (E)!

Irina on September 7, 2019

@Julie, That's correct! Keep up the good work.

Ellisw23 on April 4 at 12:49AM

Why d?

Emil-Kunkin on April 8 at 02:12AM

I would look to the parts about modesty between lines 40 and 50. The fact that women typically were not willing to go to western doctors unless they were women due to the cultural emphasis on modestly strongly suggests D.